🎧 New: AI-Generated Podcasts Turn your study notes into engaging audio conversations. Learn more

Stuvia-3314274-test-bank-for-primary-care-interprofessional-collaborative-practice-6th-edition-by-buttaro-9780323570152-chapter-1-228-complete-questions-and-answers-a.pdf

Loading...
Loading...
Loading...
Loading...
Loading...
Loading...
Loading...

Full Transcript

Test Bank For Primary Care Interprofessional Collaborative Practice 6th Edition By Buttaro 9780323570152 Chapter 1-228 Complete Questions And Answers A+ written by EasyNursingTest www.stuvia.com Downloaded by: dwadsw2 | [email protected] Distribution of this document is illegal Want to earn $1.236 ext...

Test Bank For Primary Care Interprofessional Collaborative Practice 6th Edition By Buttaro 9780323570152 Chapter 1-228 Complete Questions And Answers A+ written by EasyNursingTest www.stuvia.com Downloaded by: dwadsw2 | [email protected] Distribution of this document is illegal Want to earn $1.236 extra per year? Stuvia.com - The Marketplace to Buy and Sell your Study Material FULL TEST BANK Primary Care: A Collaborative Practice 6th Edition By Terry Buttaro PRINTED PDF | ORIGINAL DIRECTLY FROM THE PUBLISHER | 100% VERIFIED ANSWERS | DOWNLOAD IMMEDIATELY AFTER THE ORDER Complete Test bank, All Chapters are included. For more Test banks, ATI, HESI exams, and more contact us. Downloaded by: dwadsw2 | [email protected] Distribution of this document is illegal Want to earn $1.236 extra per year? Stuvia.com - The Marketplace to Buy and Sell your Study Material Table of content Chapter 1 Interprofessional Collaborative Practice Chapter 2 Translating Research Into Clinical Practice Chapter 3 Empowering Patients as Collaborative Partners Chapter 4 Coordinated Chronic Care Chapter 5 An Introduction to Health Care Disparities and Culturally Responsive Primary Care Chapter 6 Patient/Family Education and Health Literacy Chapter 7 Genetic Considerations in Primary Care Chapter 8 Risk Management Chapter 9 Adolescent Issues Chapter 10 LGBTQ Patient Care Chapter 11 Pregnancy, Prenatal Care, and Lactation Chapter 12 Human Trafficking Chapter 13 Aging and Common Geriatric Syndromes Chapter 14 Palliative Care Chapter 15 Acute, Chronic, Oncologic, and End-of-Life Pain Management in Primary Care Chapter 16 Wellness Chapter 17 Obesity and Weight Management Chapter 18 Principles of Occupational and Environmental Health in Primary Care Chapter 19 College Health Chapter 20 Presurgical Clearance Chapter 21 Preparticipation Sports Physical Chapter 22 Acute Bronchospasm Chapter 23 Anaphylaxis Chapter 24 Bites and Stings Chapter 25 Bradycardia and Tachycardia Chapter 26 Chemical Exposure Chapter 27 Electrical Injuries Chapter 28 Environmental and Food Allergies Chapter 29 Head Trauma Chapter 30 Hypotension Chapter 31 Poisoning Chapter 32 Sexual Assault Chapter 33 Syncope Chapter 34 Thermal Injuries Chapter 35 Examination of the Skin and Approach to Diagnosis of Skin Disorders Chapter 36 Surgical Office Procedures Chapter 37 Principles of Dermatologic Therapy Chapter 38 Screening for Skin Cancer Chapter 39 Adnexal Disease Chapter 40 Alopecia Chapter 41 Animal and Human Bites Chapter 42 Benign Skin Lesions Chapter 43 Bullous Pemphigoid Chapter 44 Burns (Minor) Chapter 45 Cutaneous Adverse Drug Reactions Chapter 46 Eczematous Dermatitis Chapter 47 Infections and Infestations Chapter 48 Nail Disorders Chapter 49 Maculopapular Skin Disorders Downloaded by: dwadsw2 | [email protected] Distribution of this document is illegal Want to earn $1.236 extra per year? Stuvia.com - The Marketplace to Buy and Sell your Study Material Chapter 50 Pigmentation Changes Chapter 51 Wound Management Chapter 52 Evaluation of the Eyes Chapter 53 Cataracts Chapter 54 Blepharitis, Hordeolum, and Chalazion Chapter 55 Conjunctivitis Chapter 56 Corneal Surface Defects and Ocular Surface Foreign Bodies Chapter 57 Dry Eye Syndrome Chapter 58 Nasolacrimal Duct Obstruction and Dacryocystitis Chapter 59 Preseptal and Orbital Cellulitis Chapter 60 Pingueculae and Pterygia Chapter 61 Traumatic Ocular Disorders Chapter 62 Auricular Disorders Chapter 63 Cerumen Impaction Chapter 64 Cholesteatoma Chapter 65 Impaired Hearing Chapter 66 Inner Ear Disturbances Chapter 67 Otitis Externa Chapter 68 Otitis Media Chapter 69 Tympanic Membrane Perforation Chapter 70 Chronic Nasal Congestion and Discharge Chapter 71 Epistaxis Chapter 72 Nasal Trauma Chapter 73 Rhinitis Chapter 74 Sinusitis Chapter 75 Smell and Taste Disturbances Chapter 76 Tumors and Polyps of the Nose Chapter 77 Dental Abscess Chapter 78 Diseases of the Salivary Gland Chapter 79 Epiglottitis Chapter 80 Oral Infections Chapter 81 Parotitis Chapter 82 Peritonsillar Abscess Chapter 83 Pharyngitis and Tonsillitis Chapter 84 Acute Bronchitis Chapter 85 Asthma Chapter 86 Chest Pain (Noncardiac) Chapter 87 Chronic Cough Chapter 88 Chronic Obstructive Pulmonary Disease Chapter 89 Dyspnea Chapter 90 Hemoptysis Chapter 91 Lung Cancer Chapter 92 Pleural Effusions and Pleurisy Chapter 93 Pneumonia Chapter 94 Pneumothorax Chapter 95 Pulmonary Embolism Chapter 96 Pulmonary Hypertension Chapter 97 Sarcoidosis Chapter 98 Cardiac Diagnostic Testing Chapter 99 Abdominal Aortic Aneurysm Chapter 100 Cardiac Arrhythmias Chapter 101 Carotid Artery Disease Downloaded by: dwadsw2 | [email protected] Distribution of this document is illegal Want to earn $1.236 extra per year? Stuvia.com - The Marketplace to Buy and Sell your Study Material Downloaded by: dwadsw2 | [email protected] Distribution of this document is illegal Want to earn $1.236 extra per year? Stuvia.com - The Marketplace to Buy and Sell your Study Material Chapter 102 Chest Pain and Coronary Artery Disease Chapter 103 Heart Failure Chapter 104 Hypertension Chapter 105 Infective Endocarditis Chapter 106 Myocarditis Chapter 107 Peripheral Arterial and Venous Insufficiency Chapter 108 Valvular Heart Disease and Cardiac Murmurs Chapter 109 Abdominal Pain and Infections Chapter 110 Anorectal Complaints Chapter 111 Cholelithiasis and Cholecystitis Chapter 112 Cirrhosis Chapter 113 Constipation Chapter 114 Diarrhea, Noninfectious Chapter 115 Diverticular Disease Chapter 116 Oropharyngeal Dysphagia in Adults Chapter 117 Gastroesophageal Reflux Disease Chapter 118 Gastrointestinal Hemorrhage Chapter 119 Hepatitis Chapter 120 Inflammatory Bowel Disease Chapter 121 Irritable Bowel Syndrome Chapter 122 Jaundice Chapter 123 Nausea and Vomiting Chapter 124 Pancreatitis Chapter 125 Tumors of the Gastrointestinal Tract Chapter 126 Peptic Ulcer Disease Chapter 127 Incontinence Chapter 128 Prostate Cancer Chapter 129 Prostatic Hyperplasia (Benign) Chapter 130 Proteinuria and Hematuria Chapter 131 Renal Failure Chapter 132 Sexual Dysfunction (Male) Chapter 133 Testicular Disorders Chapter 134 Urinary Calculi Chapter 135 Urinary Tract Infections and Sexually Transmitted Infections Chapter 136 Uropathies (Obstructive) and Tumors of the Genitourinary Tract (Kidneys, Ureters, and Bladder) Chapter 137 Amenorrhea Chapter 138 Bartholin Gland Cysts and Abscesses Chapter 139 Breast Disorders Chapter 140 Chronic Pelvic Pain Chapter 141 Abnormal Uterine Bleeding Chapter 142 Dysmenorrhea Chapter 143 Dyspareunia Chapter 144 Ectopic Pregnancy Chapter 145 Fertility Control Chapter 146 Genital Tract Cancers Chapter 147 Infertility Chapter 148 Menopause Chapter 149 Cervical Cancer Screening Abnormalities Chapter 150 Pelvic Inflammatory Disease Chapter 151 Sexual Dysfunction (Female) Chapter 152 Unplanned Pregnancy Chapter 153 Vulvar and Vaginal Disorders Downloaded by: dwadsw2 | [email protected] Distribution of this document is illegal Want to earn $1.236 extra per year? Stuvia.com - The Marketplace to Buy and Sell your Study Material Chapter 155 Bone Lesions Chapter 156 Bursitis Chapter 157 Fibromyalgia and Myofascial Pain Syndrome Chapter 158 Gout Chapter 159 Septic Arthritis Chapter 160 Low Back Pain Chapter 161 Hip Pain Chapter 162 Knee Pain Chapter 163 Metabolic Bone Disease Chapter 164 Neck Pain Chapter 165 Osteoarthritis Chapter 166 Osteomyelitis Chapter 167 Shoulder Pain Chapter 168 Sprains, Strains, and Fractures Chapter 169 Elbow Pain Chapter 170 Hand and Wrist Pain Chapter 171 Amyotrophic Lateral Sclerosis Chapter 172 Bell Palsy Chapter 173 Cerebrovascular Events Chapter 174 Dementia Chapter 175 Dizziness and Vertigo Chapter 176 Guillain-Barré Syndrome Chapter 177 Headache Chapter 178 Infections of the Central Nervous System Chapter 179 Movement Disorders and Essential Tremor Chapter 180 Multiple Sclerosis Chapter 181 Parkinson Disease Chapter 182 Seizure Disorder Chapter 183 Trigeminal Neuralgia Chapter 184 Intracranial Tumors Chapter 185 Adrenal Gland Disorders Chapter 186 Diabetes Mellitus Chapter 187 Hirsutism Chapter 188 Hypercalcemia and Hypocalcemia Chapter 189 Hyperkalemia and Hypokalemia Chapter 190 Hypernatremia and Hyponatremia Chapter 191 Lipid Disorders Chapter 192 Metabolic Syndrome Chapter 193 Parathyroid Gland Disorders Chapter 194 Thyroid Disorders Chapter 195 Polymyalgia Rheumatica and Giant Cell Arteritis Chapter 196 Raynaud Phenomenon Chapter 197 Rheumatoid Arthritis Chapter 198 Seronegative Spondyloarthropathies Chapter 199 Systemic Lupus Erythematosus Chapter 200 Vasculitis Chapter 201 Diving-Related Maladies Chapter 202 Fatigue Chapter 203 Fever Chapter 204 Immunodeficiency Downloaded by: dwadsw2 | [email protected] Distribution of this document is illegal Want to earn $1.236 extra per year? Stuvia.com - The Marketplace to Buy and Sell your Study Material Chapter 205 Lymphadenopathy Chapter 206 Sleep Disorders Chapter 207 Unintended Weight Loss Chapter 208 Emerging and Reemerging Infectious Diseases Chapter 209 HIV Infection Chapter 210 Influenza Chapter 211 Infectious Diarrhea Chapter 212 Infectious Mononucleosis Chapter 213 Tick-Borne Diseases Chapter 214 Tuberculosis Chapter 215 Mosquito-Borne Illness Chapter 216 Anemia Chapter 217 Blood Coagulation Disorders Chapter 218 Leukemias Chapter 219 Lymphomas Chapter 220 Myelodysplastic Syndromes Chapter 221 Interprofessional Collaborative Management of the Oncology Patient Chapter 222 Basic Principles of Oncology Treatment Chapter 223 Oncology Complications, Paraneoplastic Syndromes, and Cancer Survivorship Chapter 224 Carcinoma of Unknown Primary Chapter 225 Anxiety Disorders Chapter 226 Mood Disorders Chapter 227 Substance Use Disorders Chapter 228 Other Mental Health Disorders Downloaded by: dwadsw2 | [email protected] Distribution of this document is illegal Want to earn $1.236 extra per year? Stuvia.com - The Marketplace to Buy and Sell your Study Material Chapter 01: Interprofessional Collaborative Practice: Where We Are Today Buttaro: Primary Care: A Collaborative Practice, 6th Edition MULTIPLE CHOICE 1. A small, rural hospital is part of an Accountable Care Organization (ACO) and is designated as a Level 1 ACO. What is part of this designation? a. Bonuses based on achievement of benchmarks b. Care coordination for chronic diseases c. Standards for minimum cash reserves d. Strict requirements for financial reporting ANS: A A Level 1 ACO has the least amount of financial risk and requirements, but receives shared savings bonuses based on achievement of benchmarks for quality measures and expenditures. Care coordination and minimum cash reserves standards are part of Level 2 ACO requirements. Level 3 ACOs have strict requirements for financial reporting. 2. What was an important finding of the Advisory Board survey of 2014 about primary care preferences of patients? a. Associations with area hospitals b. Costs of ambulatory care c. Ease of access to care d. The ratio of providers to patients ANS: C As part of the 2014 survey, the Advisory Board learned that patients desired 24/7 access to care, walk-in settings and the ability to be seen within 30 minutes, and care that is close to home. Associations with hospitals, costs of care, and the ratio of providers to patients were not part of these results. MULTIPLE RESPONSE 1. Which assessments of care providers are performed as part of the value-based purchasing (VBP) initiative? (Select all that apply.) a. Appraising costs per case of care for Medicare patients b. Assessing patients’ satisfaction with hospital care c. Evaluating available evidence to guide clinical care guidelines d. Monitoring mortality rates of all patients with pneumonia e. Requiring advanced IT standards and minimum cash reserves ANS: A, B, D Value-based purchasing looks at five domain areas of processes of care, including efficiency of care (cost per case), experience of care (patient satisfaction measures), and outcomes of care (mortality rates for certain conditions). Evaluation of evidence to guide clinical care is part of evidence-based practice. The requirements for IT standards and financial status are part of Accountable Care Organization standards. WWW.THENURSINGMASTERY.COM Downloaded by: dwadsw2 | [email protected] Distribution of this document is illegal Want to earn $1.236 extra per year? Stuvia.com - The Marketplace to Buy and Sell your Study Material Chapter 02: Translating Research into Clinical Practice Buttaro: Primary Care: A Collaborative Practice, 6th Edition MULTIPLE CHOICE 1. What is the purpose of Level II research? a. To define characteristics of interest of groups of patients b. To demonstrate the effectiveness of an intervention or treatment c. To describe relationships among characteristics or variables d. To evaluate the nature of relationships between two variables ANS: C Level II research is concerned with describing the relationships among characteristics or variables. Level I research is conducted to define the characteristics of groups of patients. Level II research evaluates the nature of the relationships between variables. Level IV research is conducted to demonstrate the effectiveness of interventions or treatments. 2. Which is the most appropriate research design for a Level III research study? a. Epidemiological studies b. Experimental design c. Qualitative studies d. Randomized clinical trials ANS: B The experimental design is the most appropriate design for a Level III study. Epidemiological studies are appropriate for Level II studies. Qualitative designs are useful for Level I studies. Randomized clinical trials are used for Level IV studies. 3. What is the purpose of clinical research trials in the spectrum of translational research? a. Adoption of interventions and clinical practices into routine clinical care b. Determination of the basis of disease and various treatment options c. Examination of safety and effectiveness of various interventions d. Exploration of fundamental mechanisms of biology, disease, or behavior ANS: C Clinical research trials are concerned with determining the safety and effectiveness of interventions. Adoption of interventions and practices is part of clinical implementation. Determination of the basis of disease and treatment options is part of the preclinical research phase. Exploration of the fundamental mechanisms of biology, disease, or behavior is part of the basic research stage. WWW.THENURSINGMASTERY.COM Downloaded by: dwadsw2 | [email protected] Distribution of this document is illegal Want to earn $1.236 extra per year? Stuvia.com - The Marketplace to Buy and Sell your Study Material Chapter 03: Empowering Patients as Collaborative partners: A New Model for Primary Care Buttaro: Primary Care: A Collaborative Practice, 6th Edition MULTIPLE CHOICE 1. Which statement made by a health care provider demonstrates the most appropriate understanding for the goal of a performance report? a. “This process allows me to critique the performance of the rest of the staff.” b. “Most organizations require staff to undergo a performance evaluation yearly.” c. “It is hard to be personally criticized but that’s how we learn to change.” d. “The comments should help me improve my management skills.” ANS: D The goal of the performance report is to provide guidance to staff in the areas of professional development, mentoring, and leadership development. A peer review is written by others who perform similar skills (peers). The remaining options may be true but do not provide evidence of understanding of the goal of this professional requirement. MULTIPLE RESPONSE 1. Which assessment question would a health care provider ask when engaging in the previsit stage of the new model for primary care? (Select all that apply.) a. “Are you ready to discuss some of the community resources that are available?” b. “Are you experiencing any side effects from your newly prescribed medications?” c. “Do you anticipate any problems with adhering to your treatment plan?” d. “Are you ready to discuss the results of your laboratory tests?” e. “Do you have any questions about the lab tests that have been ordered for you?” ANS: B, C, E The nursing responsibilities in the previsit stage include assessing the patient’s tolerance of prescribed medications, understanding of existing treatment plan, and education about required lab testing. The primary care provider is responsible for screening lab data and discussing community resources during the actual visit. WWW.THENURSINGMASTERY.COM Downloaded by: dwadsw2 | [email protected] Distribution of this document is illegal Want to earn $1.236 extra per year? Stuvia.com - The Marketplace to Buy and Sell your Study Material Chapter 04: Coordinated Chronic Care Buttaro: Primary Care: A Collaborative Practice, 6th Edition MULTIPLE CHOICE 1. To reduce adverse events associated with care transitions, the Centers for Medicare and Medicaid Service have implemented which policy? a. Mandates for communication among primary caregivers and hospitalists b. Penalties for failure to perform medication reconciliations at time of discharge c. Reduction of payments for patients readmitted within 30 days after discharge d. Requirements for written discharge instructions for patients and caregivers ANS: C As a component of the Affordable Care Act, the Centers for Medicare and Medicaid Service developed the Readmissions Reduction Program reducing payments for certain patients readmitted within 30 days of discharge. The CMS did not mandate communication, institute penalties for failure to perform medication reconciliations, or require written discharge instructions. 2. According to multiple research studies, which intervention has resulted in lower costs and fewer rehospitalizations in high-risk older patients? a. Coordination of posthospital care by advanced practice health care providers b. Frequent posthospital clinic visits with a primary care provider c. Inclusion of extended family members in the outpatient plan of care d. Telephone follow-up by the pharmacist to assess medication compliance ANS: A Research studies provided evidence that high-risk older patients who had posthospital care coordinated by an APN had reduced rehospitalization rates. It did not include clinic visits with a primary care provider, inclusion of extended family members in the plan of care, or telephone follow-up by a pharmacist. MULTIPLE RESPONSE 1. Which advantages are provided to the chronically ill patient by personal electronic monitoring devices? (Select all that apply.) a. Helps provide more patient control their health and lifestyle b. Eliminates need for regular medical and nursing follow-up visits c. Helps the early identification of patient health-related problems d. Helps health care providers in keeping track of the patient’s health status e. Cost is often covered by Medicare ANS: A, C, D, E WWW.THENURSINGMASTERY.COM Downloaded by: dwadsw2 | [email protected] Distribution of this document is illegal Want to earn $1.236 extra per year? Stuvia.com - The Marketplace to Buy and Sell your Study Material The explosion in availability of personal electronic monitoring devices is potentially useful to many patients with chronic disease and others hoping to maintain good health. Data is recorded and can help people have more control over their health and lifestyle. It can also help health care providers keep track of their patients’ health status, as information from these devices can be uploaded into Apps and electronic health records. These devices are becoming more affordable and some are covered by Medicare. Coupled with telehealth, e-mail, or other electronic communication with health care providers allows patient problems to be recognized early. Medical and nursing follow-up is still required as problems arise. WWW.THENURSINGMASTERY.COM Downloaded by: dwadsw2 | [email protected] Distribution of this document is illegal Want to earn $1.236 extra per year? Stuvia.com - The Marketplace to Buy and Sell your Study Material Chapter 05: An Introduction to Health Care Disparities and Culturally Responsive Primary Care Buttaro: Primary Care: A Collaborative Practice, 6th Edition MULTIPLE CHOICE 1. A primary care provider administers the “Newest Vital Sign” health literacy test to a patient newly diagnosed with a chronic disease. What information is gained by administering this test? a. Ability to calculate data, along with general knowledge about health b. Ease of using technology and understanding of graphic data c. Reading comprehension and reception of oral communication d. Understanding of and ability to discuss health care concerns ANS: A The “Newest Vital Sign” tests asks patients to look at information on an ice cream container label and answer questions that evaluate ability to calculate caloric data and to grasp general knowledge about food allergies. It does not test understanding of technology or directly measure reading comprehension. It does not assess oral communication. The “Ask Me 3” tool teaches patients to ask three primary questions about their health care and management. 2. What is the main reason for using the REALM-SF instrument to evaluate health literacy? a. It assesses numeracy skills. b. It enhances patient–provider communication. c. It evaluates medical word recognition. d. It measures technology knowledge. ANS: C The Rapid Estimate of Adult Literacy in Medicine–Short Form (REALM-SF) is an easy and fast tool that measures medical word recognition. It does not evaluate numeracy. The “Ask Me 3” tool enhances patient–provider communication. This tool does not evaluate understanding of technology. 3. A female patient who is from the Middle East schedules an appointment in a primary care office. To provide culturally responsive care, what will the clinic personnel do when meeting this patient for the first time? a. Ensure that she is seen by a female provider. b. Include a male family member in discussions about health care. c. Inquire about the patient’s beliefs about health and treatment. d. Research middle eastern cultural beliefs about health care. ANS: C It is important not to make assumptions about beliefs and practices associated with health care and to ask the patient about these. While certain practices are common in some cultural and ethnic groups, assuming that all members of those groups follow those norms is not culturally responsive. WWW.THENURSINGMASTERY.COM Downloaded by: dwadsw2 | [email protected] Distribution of this document is illegal Want to earn $1.236 extra per year? Stuvia.com - The Marketplace to Buy and Sell your Study Material Chapter 06: Patient/Family Education and Health Literacy Buttaro: Primary Care: A Collaborative Practice, 6th Edition MULTIPLE CHOICE 1. A primary care provider is providing care for a postsurgical client who recently immigrated to the United States and speaks English only marginally. What intervention will provide the most effective means of communicating postdischarge information to the client? a. Postpone discharge until the client is fully recovered from the surgery. b. Requesting that a family member who speaks English be present during the teaching session c. Providing the necessary information in written form in the client’s native language d. Requesting the services of a professional interpreter fluent in the client’s native language ANS: D Only approved, professional interpreters experienced in health care interpretation are appropriate interpreters for patients. Family members or friends should not be used as interpreters. Use of family members or friends may create misinterpretation or misunderstanding between the provider and the patient. Family members may not understand medical terms or may interpret only what they feel is important, or patients might feel uncomfortable divulging personal information to the person interpreting. Written information in the client’s native language may be a means of reinforcing instructions but are not a substitute of person-to-person education. It is neither realistic nor necessary to postpone discharge for this reason. MULTIPLE RESPONSE 1. What question asked by the client newly diagnosed with congestive heart failure demonstrates the effectiveness of previous education concerning the Ask Me 3 health literacy tool? (Select all that apply.) a. “Where can I get assistance with the cost of my medications?” b. “Why is it important for me to take this newly prescribed medication?” c. “Is it true that high blood pressure isn’t causing my problem?” d. “Is congestive heart failure curable with appropriate treatment?” e. “Would watching my intake of salt help me manage this problem? ANS: B, C, E While all these questions are appropriate, the Ask Me 3 tool encourages the client to question what the problem is, what they need to do to manage the problem, and why it is important to follow the treatment plan. Financial support and curability of the problem is not directly addressed by this tool. WWW.THENURSINGMASTERY.COM Downloaded by: dwadsw2 | [email protected] Distribution of this document is illegal Want to earn $1.236 extra per year? Stuvia.com - The Marketplace to Buy and Sell your Study Material Chapter 07: Genetic Considerations in Primary Care Buttaro: Primary Care: A Collaborative Practice, 6th Edition MULTIPLE CHOICE 1. A patient expresses concern that she is at risk for breast cancer. To best assess the risk for this patient, what is the best initial action? a. Ask if there is a family history of breast cancer. b. Gather and record a three-generation pedigree. c. Order a genetic test for the breast cancer gene. d. Recommend direct-to-consumer genetic testing. ANS: B The three-generation pedigree is the best way to evaluate genetic risk. Asking about a family history is not a systematic risk assessment and does not specify who in the family has the history or whether there is a pattern. Genetic testing and direct-to-consumer (DTC) genetic testing are not the initial actions when assessing genetic risk. 2. A patient asks about direct-to-consumer (DTC) genetic testing. What will the provider tell the patient? a. It is not useful for identifying genetic diseases. b. Much of the information does not predict disease risk. c. The results are shared with the patient’s insurance company. d. The results must be interpreted by a provider. ANS: B DTC testing gives a lot of information, but much of it does not contribute to disease prediction, since mutations are not necessarily related to specific diseases. The tests are useful but must be interpreted accurately. The results are confidential and do not have to be interpreted by a provider. WWW.THENURSINGMASTERY.COM Downloaded by: dwadsw2 | [email protected] Distribution of this document is illegal Want to earn $1.236 extra per year? Stuvia.com - The Marketplace to Buy and Sell your Study Material Chapter 08: Risk Management Buttaro: Primary Care: A Collaborative Practice, 6th Edition MULTIPLE CHOICE 1. What is an important part of patient care that can minimize the risk of a formal patient complaint even when a mistake is made? a. Ensuring informed consent for all procedures b. Maintaining effective patient communication c. Monitoring patient compliance and adherence d. Providing complete documentation of visits ANS: B Effective patient communication is key to building trust and rapport and ineffective communication is a predictor for malpractice claims. The other items are important aspects of care and may help the provider during the investigation of a claim, but do not minimize the risk. MULTIPLE RESPONSE 1. What are some causes for failures or delays in diagnosing patients resulting in malpractice claims? (Select all that apply.) a. Failing to recognize a medication complication b. Failing to request appropriate consultations c. Improper performance of a treatment d. Not acting on diagnostic test results e. Ordering a wrong medication ANS: B, D Failing to obtain consultations when indicated or not acting on diagnostic test results can lead to diagnosis-related failures. Failing to recognize medication complications and ordering a wrong medication lead to medication prescribing allegations. Improper performance of a treatment can lead to treatment-related malpractice claims. WWW.THENURSINGMASTERY.COM Downloaded by: dwadsw2 | [email protected] Distribution of this document is illegal Want to earn $1.236 extra per year? Stuvia.com - The Marketplace to Buy and Sell your Study Material Chapter 09: Adolescent Issues Buttaro: Primary Care: A Collaborative Practice, 6th Edition MULTIPLE CHOICE 1. Which behavior is most characteristic of early adolescence? a. Arguing with parents and teachers b. Assimilating adult roles and thinking c. Exhibiting fatigue more frequently d. Experimenting with sex and risky behaviors ANS: A Early adolescents challenge authority, experience wide mood swings, reject the ideation of childhood, and can be argumentative and disobedient. Middle adolescents experience fatigue and begin experimenting with sex and risky behaviors. Late adolescents begin to assimilate adult roles. 2. What is the initial sign of puberty in the adolescent male? a. Deepening of the voice b. Elongation of the penis c. Nocturnal emissions d. Testicular enlargement ANS: D Testicular enlargement is the initial sign of puberty in adolescent males. Penile growth and nocturnal emissions occur later as does deepening of the voice. 3. A parent reports that an adolescent child does well in school but seems to consistently make poor decisions about activities with friends. What will the practitioner recommend as an approach to help the adolescent make better decisions? a. Correcting the adolescent’s decisions and judgments b. Listening without making suggestions about choices c. Making decisions for the adolescent to provide guidance d. Providing information about appropriate behavior ANS: B Listening without correcting is the best approach to help adolescents learn to make good decisions. Correcting the decisions, making decisions for the adolescent, or giving information that is unsolicited is not recommended. WWW.THENURSINGMASTERY.COM Downloaded by: dwadsw2 | [email protected] Distribution of this document is illegal Want to earn $1.236 extra per year? Stuvia.com - The Marketplace to Buy and Sell your Study Material Chapter 10: LGBTQ Patient Care: Care of Sexual and Gender Minority People Buttaro: Primary Care: A Collaborative Practice, 6th Edition MULTIPLE CHOICE 1. What is the focus of the Minority Stress Theory? a. Clarifying the various terms used to describe the LGBTQ community b. Understanding the health needs of select members of the sexual minority community c. Helping health care providers eliminate biases in the care they provide to the members of the LGBTQ community d. Facilitating the management of stress related to the lifestyle choices made by members of sexual minority populations. ANS: B The focus of the Minority Stress Theory is to provide a framework to the understanding of the health disparities in the sexual and gender minority (SGM) communities. The theory may help achieve some aspect of the other options, but that is not its focus. 2. What is the medical diagnostic term used to identify transgender patients? a. Gender dysphoria b. Gender expression disorder c. Gender identity disorder d. Gender role unconformity ANS: A Gender dysphoria is the term used to identify transgender patients in order to justify the medical necessity of treatments for transgender patients. It replaces the previous “gender identity disorder” designation. WWW.THENURSINGMASTERY.COM Downloaded by: dwadsw2 | [email protected] Distribution of this document is illegal Want to earn $1.236 extra per year? Stuvia.com - The Marketplace to Buy and Sell your Study Material Chapter 11: Pregnancy, Prenatal Care, and Lactation Buttaro: Primary Care: A Collaborative Practice, 6th Edition MULTIPLE CHOICE 1. A woman who is currently pregnant reports that she has had three previous pregnancies: twins delivered at 35-weeks gestation (both living), one at 38-weeks gestation (living), and one miscarriage at 16-weeks gestation. How will this be recorded as her G/TPAL in her electronic medical record? a. G4P:1113 b. G4P:1213 c. G5P:1113 d. G5P:1213 ANS: A Using the notation G (number of pregnancies), T (term deliveries), P (preterm deliveries), A (abortions—elective or spontaneous), and L (living children), this patient is G4P:1113. She is in her fourth pregnancy, so is G4. She has had one delivery at 38 weeks or more, one delivery (of twins) at less than 36 weeks gestation, one spontaneous abortion, and has three living children. 2. A pregnant woman who is overweight has no previous history of hypertension (HTN) or diabetes mellitus (DM). Her initial screening exam reveals a blood pressure of 140/90 and a fasting blood glucose of 128 mg/dL. What will the practitioner do? a. Initiate insulin therapy. b. Monitor blood pressure and fasting blood glucose closely. c. Prescribe an antihypertensive medication. d. Refer the patient to a high-risk pregnancy specialist. ANS: B This woman, although she has no previous history of HTN or DM, is at elevated risk because of obesity. Her initial screening lab values are at the high end of normal, indicating potential development of gestational HTN and gestational DM. The initial response of the practitioner should be to monitor the patient closely and consider treatment at the first signs of development of these complications. Referral is warranted when these conditions become severe. 3. The mother of a 3-day-old newborn reports that her infant health care providers every 4 hours during the day and sleeps 6 hours at night. What will the provider recommend? a. Awakening the baby every 3 hours to health care provider b. Continuing this schedule until the infant is 6 months old c. Ensuring that her infant health care providers for 15 to 20 minutes each time d. Pumping her breasts to maintain her milk supply ANS: A Newborn infants should health care provider 8 to 12 times daily and mothers should be encouraged to awaken a sleepy baby to health care provider every 2 to 3 hours or more often. The feedings will gradually space out as the infant is older. WWW.THENURSINGMASTERY.COM Downloaded by: dwadsw2 | [email protected] Distribution of this document is illegal Want to earn $1.236 extra per year? Stuvia.com - The Marketplace to Buy and Sell your Study Material 4. An infant who has just begun nursing develops hyperbilirubinemia. What will the provider tell the mother? a. To decrease the frequency of breastfeeding b. To supplement feedings with extra water c. To switch to formula until the bilirubin level drops d. To use a breast pump to increase her milk supply ANS: D Infants with suboptimal breastfeeding can have starvation jaundice and mothers should be encouraged to increase the frequency of breastfeeding and should be offered a breast pump to increase milk supply. It is not recommended to supplement with water or sugar water or to switch to formula. 5. A mother who has been breastfeeding her infant for several weeks develops a fever, breast warmth, and breast tenderness. What will the provider recommend? a. Ice packs and decreased frequency of nursing b. Ice packs and increased frequency of nursing c. Warm packs and decreased frequency of nursing d. Warm packs and increased frequency of nursing ANS: D This mother has symptoms of mastitis. She should be encouraged to use warm packs for comfort and to increase the frequency of nursing to relieve the pressure. MULTIPLE RESPONSE 1. A pregnant woman reports not having had any vaccinations as a child but requests vaccines during her pregnancy. Which vaccines may be given? (Select all that apply.) a. Human papillomavirus (HPV) b. Inactivated influenza c. Live, attenuated influenza d. Measles, mumps, and rubella (MMR) e. Tetanus, diphtheria, and acellular pertussis (Tdap) f. Varicella ANS: A, B, E Tdap is recommended to pregnant woman, optimally between 27- and 36-weeks gestation. Inactivated is strongly recommended and may be given at any point in the pregnancy. Hepatitis B is given to women at risk if needed. Live, attenuated influenza vaccine, MMR, and varicella vaccines are not recommended during pregnancy. WWW.THENURSINGMASTERY.COM Downloaded by: dwadsw2 | [email protected] Distribution of this document is illegal Want to earn $1.236 extra per year? Stuvia.com - The Marketplace to Buy and Sell your Study Material Chapter 12: Human Trafficking Buttaro: Primary Care: A Collaborative Practice, 6th Edition MULTIPLE CHOICE 1. An unaccompanied teenager is being treated in the emergency department for stomach pains. Which statement would alert the health care provider to the possibility that the patient may be a victim of human trafficking? a. “I can’t pay to see a doctor.” b. “I’ve never been to a hospital before.” c. “You are being very nice to me.” d. “Please, please, don’t hurt me.” ANS: D Victims of human trafficking may display a profound fearfulness during examination often focusing on being hurt. None of the other options provide such a trigger for suspicion since poverty and unfamiliarity with established health care providers is not uncommon. 2. A health care provider strongly suspects that the patient being treated for a laceration to the forehead may be a victim of human trafficking. What intervention should be implemented initially before proceeding with a complete screening? a. Determine the patient’s ability to consent to treatment. b. Begin cleansing the wound in preparation for suturing. c. Transfer the patient to a private treatment room. d. Notify the police of the situation. ANS: C First and foremost, the provider will want to provide a private location and to make sure the patient is alone before asking any questions. The remaining options can wait to be initiated until privacy is provided. MULTIPLE RESPONSE 1. What assessment data would trigger the health care provider’s suspicion that the patient seen in the emergency department may be a victim of human trafficking? (Select all that apply.) a. Provides details related to cause of injury b. Appears to be illiterate c. Has “$50” tattooed on the left shoulder blade d. Cannot provide a local address of residence e. Looks much younger than state age of 21 ANS: B, C, D, E Victims of sex trafficking may appear younger than their stated age, demonstrate learning disabilities, and have unusual tattoos. Often, the victim will not be able to provide their address, identification documents, and they can be unaware of their location or date. Such a patient is likely to present vague or inconsistent stories related to their illness or injury. WWW.THENURSINGMASTERY.COM Downloaded by: dwadsw2 | [email protected] Distribution of this document is illegal Want to earn $1.236 extra per year? Stuvia.com - The Marketplace to Buy and Sell your Study Material Chapter 13: Aging and Common Geriatric Syndromes Buttaro: Primary Care: A Collaborative Practice, 6th Edition MULTIPLE CHOICE 1. What intervention will the provider implement when prescribing medications to an 80-year-old patient? a. Beginning with higher doses and decrease according to the patient’s response b. Consulting the Beers list to help identify potentially problematic drugs c. Ensuring that the patient does not take more than five concurrent medications d. Reviewing all patient medications at the annual health maintenance visit ANS: B The Beers list provides a list of potentially inappropriate medications in all patients aged 65 and older and helps minimize drug-related problems in this age group. Older patients should be started on lower doses with gradual increase of doses depending on response and side effects. Patients who take five or more drugs are at increased risk for problems of polypharmacy, but many will need to take more than five drugs; providers must monitor their response more closely. Medications should be reviewed at all visits, not just annually. 2. An 80-year-old woman who lives alone is noted to have a recent weight loss of 5 pounds. She appears somewhat confused, according to her daughter, who is concerned that she is developing dementia. The provider learns that the woman still drives, volunteers at the local hospital, and attends a book club with several friends once a month. What is the initial step in evaluating this patient? a. Obtaining a CBC, serum electrolytes, BUN, and glucose b. Ordering a CBC, serum ferritin, and TIBC c. Referring the patient to a dietician for nutritional evaluation d. Referring the patient to a neurologist for evaluation for AD ANS: A Patients with weight loss, confusion, and lethargy are often dehydrated and this should be evaluated by looking at Hgb and Hct, electrolytes, and BUN. This patient is currently leading an active life, so the likelihood that recent symptoms are related to AD, although this may be evaluated if dehydration is ruled out. Anemia would be a consideration when dehydration is ruled out. Referrals are not necessary unless initial evaluations suggest that malnutrition or AD is present. MULTIPLE RESPONSE 1. The practitioner is establishing a plan for routine health maintenance for a new client who is 80 years old. The client has never smoked and has been in good health. What will the practitioner include in routine care for this patient? (Select all that apply.) a. Annual hypertension screening b. Baseline abdominal aorta ultrasound c. Colonoscopy every 10 years d. One-time hepatitis B vaccine e. Pneumovax vaccine if not previously given f. Yearly influenza vaccine WWW.THENURSINGMASTERY.COM Downloaded by: dwadsw2 | [email protected] Distribution of this document is illegal Want to earn $1.236 extra per year? Stuvia.com - The Marketplace to Buy and Sell your Study Material ANS: E, F For older clients a one-time pneumovax is given after age 65. Influenza vaccine should be given every year. Hypertension screening should be performed at each office visit, not just annually. An abdominal aorta US is performed once for every smoking male. Colonoscopy is performed every 10 years after age 50, but not after age 74. WWW.THENURSINGMASTERY.COM Downloaded by: dwadsw2 | [email protected] Distribution of this document is illegal Want to earn $1.236 extra per year? Stuvia.com - The Marketplace to Buy and Sell your Study Material Chapter 14: Palliative Care Buttaro: Primary Care: A Collaborative Practice, 6th Edition MULTIPLE CHOICE 1. When should palliative care be initiated by a primary care provider? a. After an ill patient asks for Hospice services b. As part of routine health maintenance c. When a patient is diagnosed with a serious disease d. When an interdisciplinary team is formed to manage a disease ANS: B Palliative care support begins with an understanding of a patient’s preferences and helping the patient to identify goals of care. Health care providers should initiate such discussions as a component of the initial history of adults regardless of age or health status. Palliative care services may be ordered when a patient is diagnosed with a serious disease; waiting until the patient asks for Hospice services or when an interdisciplinary team is formed increases the chances of providing end-of-life care that does not meet the patient’s needs. MULTIPLE RESPONSE 1. When using the “Five Wishes” approach to documenting patient preferences for end-of-life care, the provider will document which types of preferences? (Select all that apply.) a. A directive to avoid calling 911 at the time of death b. A specific list of treatments the patient does not want c. How much information to give various family members d. The level of sedation versus alertness the patient desires e. The people designated to make care decisions for the patient ANS: C, D, E The Five Wishes approach addresses the type of care a patient wants as a disease progresses and is less defensive than the traditional advance directive which indicates the type of care a patient does not want. Calling 911 may be done without requiring resuscitation if the patient has an appropriate advanced directive in place. 2. A patient who is near death is exhibiting signs of agitation, anxiety, and intractable pain. When discussing palliative sedation with this patient’s family, what will be discussed? (Select all that apply.) a. The chance that refractory symptoms will be alleviated b. The fact that this is an intervention of last resort c. The likelihood that the patient will develop dependence on the drugs d. The need for informed consent from the patient and family e. The possibility that this measure may hasten death ANS: B, D, E WWW.THENURSINGMASTERY.COM Downloaded by: dwadsw2 | [email protected] Distribution of this document is illegal Want to earn $1.236 extra per year? Stuvia.com - The Marketplace to Buy and Sell your Study Material Palliative sedation is used as a treatment of last resort for patients whose symptoms are intolerable or refractory. Patients, if possible, and family members must give informed consent. This treatment has the possibility of hastening death by inhibiting respirations. Symptoms will not be alleviated by using the measure. The chance of drug dependence is irrelevant in this situation. WWW.THENURSINGMASTERY.COM Downloaded by: dwadsw2 | [email protected] Distribution of this document is illegal Want to earn $1.236 extra per year? Stuvia.com - The Marketplace to Buy and Sell your Study Material Chapter 15: Acute, Chronic, Oncologic, and End-of-Life Pain Management in Primary Care Buttaro: Primary Care: A Collaborative Practice, 6th Edition MULTIPLE CHOICE 1. A patient who has chronic lower back pain reports increased difficulty sleeping unrelated to discomfort, along with a desire to quit working. What will the provider do? a. Ask the patient about addiction issues. b. Consult with a social worker. c. Increase the dosage of prescribed pain medications. d. Order radiographic studies of the lower spine. ANS: B Patients who exhibit poor sleep and poor coping may be developing mental defeat as a result of chronic pain and should be evaluated and treated early for this to prevent further disability and improve functionality. Substance abuse may be a part of mental defeat and should be evaluated based on assessment findings. Unless the symptoms are related to pain, increasing the dose of analgesics and ordering diagnostic studies are not indicated. 2. A patient with chronic leg pain describes the pain as “stabbing” and “throbbing.” This is characteristic of which type of pain? a. Neuropathic pain b. Referred pain c. Somatic pain d. Visceral pain ANS: C Somatic pain is caused by the activation of nociceptors in the peripheral tissues, including skin, bones, muscles, and soft tissue and is usually well-localized and characterized as stabbing, aching, or throbbing. Neuropathic pain occurs from injury to or disease of the nervous system and is described as burning, shooting, or tingling. Referred pain is a kind of visceral pain that is localized, but not attributable to the involved organ. Visceral pain is related to an organ and is often referred and poorly localized. 3. A patient is beginning treatment for chronic pain and is unable to tolerate nonsteroidal anti-inflammatory drugs. What will the provider prescribe for this patient? a. A mixed opiate product b. A pure opioid compound c. A referral for a nerve block procedure d. A selective serotonin reuptake inhibitor (SSRI) ANS: D Using the three-step analgesic ladder, the provider should use step 1 medications that include NSAIDs, tricyclic antidepressants, selective serotonin reuptake inhibitors, or anticonvulsants. Since the patient cannot tolerate NSAIDs, an SSRI is an appropriate choice. The next step if these fail is a mixed opioid product. The third step is a pure opioid product. If medication therapy fails, a referral for nerve block may be necessary. WWW.THENURSINGMASTERY.COM Downloaded by: dwadsw2 | [email protected] Distribution of this document is illegal Want to earn $1.236 extra per year? Stuvia.com - The Marketplace to Buy and Sell your Study Material Chapter 16: Wellness: An Integrated Perspective Buttaro: Primary Care: A Collaborative Practice, 6th Edition MULTIPLE CHOICE 1. An international traveler plans to travel to Kenya in sub-Saharan Africa. Which is an important disease precaution for this person? a. Carrying chloroquine to take as needed b. Starting prophylactic doxycycline before travel c. Taking precautions against Chikungunya fever d. Understanding how Ebola virus is transmitted ANS: B Malaria is a greater concern worldwide than Ebola virus and Chikungunya fever, although both are emerging diseases. Travelers should take antimalarial medications and, in this part of Africa, where there is widespread resistance to chloroquine, doxycycline is a better choice as prophylaxis. Even when chloroquine is appropriate, it must be taken prior to travel and not as needed. Chikungunya fever is a disease of the Western Hemisphere, especially in the Caribbean. Ebola virus is epidemic in western Africa and not in Kenya. 2. A patient who is planning international travel to a developing country asks the provider about vaccinations. Which is true about pretravel vaccines? a. Country-specific guidelines are provided by individual embassies. b. Malaria vaccine is the most important vaccine for worldwide travel. c. Requirements should be reviewed at least 4 to 6 weeks prior to travel. d. There are at least five required vaccines for entry into certain countries. ANS: C Patients seeking immunizations prior to international travel should have these reviewed at least 4 to 6 weeks prior so that antibody responses and completion of vaccine series may occur. Country-specific guidelines may be found on the CDC website. Malaria is not prevented by vaccine, but by prophylactic antimalarial drugs. There are only two vaccines that are required. 3. What is a goal of the Healthy People initiative? a. To increase a patient’s quality of life b. To create physical environments that promote proper health c. To achieve health equality by eliminating disparities d. To provide free health care to those unable to pay for care e. To promote healthy behaviors across all life stages ANS: C Overarching goals of the Healthy People initiative are to increase quality and length of life, free of preventable disease, disability, injury, and premature death; to achieve health equality by eliminating disparities; to create social and physical environments that promote proper health; and to promote increased quality of life, healthy development, and healthy behaviors across all life stages, all goals consistent with the definition of wellness. Free health care is not a stated goal of the initiative. WWW.THENURSINGMASTERY.COM Downloaded by: dwadsw2 | [email protected] Distribution of this document is illegal Want to earn $1.236 extra per year? Stuvia.com - The Marketplace to Buy and Sell your Study Material Chapter 17: Obesity and Weight Management Buttaro: Primary Care: A Collaborative Practice, 6th Edition MULTIPLE CHOICE 1. A woman who is obese has a neck circumference of 16.5 cm. Which test is necessary to assess for complications of obesity in this patient based on this finding? a. Electrocardiography b. Gallbladder ultrasonography c. Mammography d. Polysomnography ANS: D Women with a neck circumference greater than 16 cm have an increased risk of obstructive sleep apnea and should have polysomnography to assess for this complication. The other tests may be necessary for obese patients but are not specific to this finding. MULTIPLE RESPONSE 1. Which medications are associated with weight gain? (Select all that apply.) a. Antibiotics b. Antidepressants c. Antihistamines d. Insulin analogs e. Anticonvulsants ANS: B, C, D, E Antidepressants, antihistamines, insulin and insulin analogs, and seizure medications are all associated with weight gain. Antibiotics are not associated with weight gain. WWW.THENURSINGMASTERY.COM Downloaded by: dwadsw2 | [email protected] Distribution of this document is illegal Want to earn $1.236 extra per year? Stuvia.com - The Marketplace to Buy and Sell your Study Material Chapter 18: Principles of Occupational and Environmental Health in Primary Care Buttaro: Primary Care: A Collaborative Practice, 6th Edition MULTIPLE CHOICE 1. A patient who has a history of working around asbestos and silica fibers is concerned about developing lung disease. The primary care provider determines that the patient has a previous history of asthma as a child and currently has frequent episodes of bronchitis. A physical examination is normal and pulmonary function tests and radiographs are negative. What action is correct? a. Reassure the patient about the normal findings. b. Refer the patient to an occupational health specialist. c. Request a workplace environmental assessment. d. Suggest that the patient follow up with a pulmonologist. ANS: B Patients with environmental exposure may not have symptoms or positive findings. Because this patient reports frequent bronchitis, this should be followed up with an occupational health specialist who can evaluate the degree of exposure and perform further testing. Normal findings are not necessarily reassuring. The occupational specialist may request an environmental assessment. Pulmonologists are not trained in occupational health. MULTIPLE RESPONSE 1. During a preplacement screening for a person hired for a job requiring heavy lifting, a primary care provider notes that the new employee has environmental allergies, a history of gastroesophageal reflux disease (GERD), recurrent eczema, a previous history of an ankle fracture, and normal lower back strength and flexibility. A urine drug screen is negative. What will be included in the report to the employer? (Select all that apply.) a. GERD history b. History of allergies and eczema c. History of ankle fracture d. Lower back screening results e. Urine drug screening results ANS: D, E Only findings related to the ability of the individual to perform position requirements for the job are included in the report. Other findings should not be included, even though they may need to be addressed. WWW.THENURSINGMASTERY.COM Downloaded by: dwadsw2 | [email protected] Distribution of this document is illegal Want to earn $1.236 extra per year? Stuvia.com - The Marketplace to Buy and Sell your Study Material Chapter 19: College Health Buttaro: Primary Care: A Collaborative Practice, 6th Edition MULTIPLE CHOICE 1. A female college student seeks information about emergency contraception. What is the most important part of the assessment of this patient? a. Cultural considerations for use of contraception b. Feelings of guilt about a possible pregnancy c. Possible concerns about confidentiality d. The female’s sense of control in sexual situations ANS: D Because college women are at greater risk for sexual violence and assault, a request for emergency contraception must be followed by an evaluation of possible rape or assault. The other considerations may be part of the assessment but are not as important as determining whether a rape has occurred. 2. A female freshman college student tells the primary care provider at the student health center that she has a history of anorexia nervosa that has been well-controlled for several years. What will the provider recommend for this student? a. Dietary counseling b. Participation in sports c. Regular weight assessments d. Stress management strategies ANS: D Students with previous eating disorders may regress when stressed, so stress management is essential. Unless she begins to regress, dietary counseling is not indicated. Many who participate in sports will develop eating disorders to control weight. It is not necessary to evaluate weight regularly. WWW.THENURSINGMASTERY.COM Downloaded by: dwadsw2 | [email protected] Distribution of this document is illegal Want to earn $1.236 extra per year? Stuvia.com - The Marketplace to Buy and Sell your Study Material Chapter 20: Presurgical Clearance Buttaro: Primary Care: A Collaborative Practice, 6th Edition MULTIPLE CHOICE 1. Which patient should have pulmonary function testing as part of the presurgical exam? a. A patient older than 60 years of age b. A patient undergoing major intrathoracic surgery c. A patient with a history of pneumonia in the last 2 years d. A patient with diabetes and morbid obesity ANS: B Any patient undergoing major thoracic surgery should have pulmonary function testing. Age over 60 years, a history of pneumonia, and diabetes and obesity do not require pulmonary function testing unless there is comorbid COPD. MULTIPLE RESPONSE 1. Which factors determine which diagnostic tests should be performed in a presurgical clearance evaluation? (Select all that apply.) a. Patient’s age b. Patient’s comorbidities c. Previous surgeries d. Surgeon’s preference e. Type of anesthetic agent planned ANS: A, B, D, E The patient’s age and comorbidities, surgeon preference, and the type of anesthetic planned all determine which presurgical diagnostic tests will be performed. The patient’s previous surgeries do not determine presurgical testing. WWW.THENURSINGMASTERY.COM Downloaded by: dwadsw2 | [email protected] Distribution of this document is illegal Want to earn $1.236 extra per year? Stuvia.com - The Marketplace to Buy and Sell your Study Material Chapter 21: Preparticipation Sports Physical Buttaro: Primary Care: A Collaborative Practice, 6th Edition MULTIPLE CHOICE 1. During a preparticipation sports physical, the examiner notes a difference in strength of the patient’s radial and femoral pulses with the femoral pulses being weaker. What will the provider do? a. Evaluate for orthostatic hypotension. b. Obtain Doppler studies of lower extremity circulation. c. Reassure the patient that this is a normal finding. d. Refer the patient for a cardiologic exam. ANS: D Differences in strength between radial and femoral pulses may indicate coarctation of the aorta and should be evaluated by a cardiologist. This finding does not indicate orthostatic hypotension. The likelihood of decreased circulation is low in a young athlete. 2. A high-school adolescent is being screened for fitness before participating in sports. The adolescent has a normal examination and the examiner notes S1 and S2 heart sounds without murmur, normal blood pressure, and equal pulses. The parent reports that the adolescent’s father has a history or Wolff-Parkinson-White syndrome, which has been treated. What will the provider do? a. Clear the adolescent to play sports. b. Perform an electrocardiogram. c. Refer the adolescent to a cardiologist. d. Tell the adolescent that sports are not allowed. ANS: C A positive family history of Wolff-Parkinson-White syndrome requires physician consultation or referral before medical clearance can be given. The adolescent has a normal heart rate and physical exam, so the ECG may not yield significant or useful results. The examiner cannot clear the adolescent without consulting with a specialist. The adolescent may be cleared for sports by the specialist. 3. An overweight adolescent who takes metformin has type 2 diabetes with a HgA1C of 8.5% and asks about sports participation. What will the provider recommend? a. Losing weight prior to initiating sports participation b. Participation in strenuous sports to help with weight loss c. Referral to the endocrinologist for sports clearance d. Switching to insulin therapy prior to participation ANS: C Patients with poorly controlled diabetes should be referred to a specialist prior to clearance for sports participation. This patient has an elevated HgA1C, indicating poor control. The endocrinologist may suggest the other options, but the primary care provider should not clear this patient for participation in sports. WWW.THENURSINGMASTERY.COM Downloaded by: dwadsw2 | [email protected] Distribution of this document is illegal Want to earn $1.236 extra per year? Stuvia.com - The Marketplace to Buy and Sell your Study Material Chapter 22: Acute Bronchospasm Buttaro: Primary Care: A Collaborative Practice, 6th Edition MULTIPLE CHOICE 1. A patient diagnosed with asthma has been prescribed three bronchodilator treatments but continues to experience wheezing and shortness of breath. The health care provider caring for the patient notes an oxygen saturation of 90% on room air. What action is indicated? a. Administer oxygen and continue to monitor the patient. b. Contact the respiratory therapist to administer another treatment. c. Notify the patient’s physician immediately. d. Reassure the patient that the treatments will take effect soon. ANS: C Patients with bronchospasm who have oxygen saturations less than 92% on room air and who fail to improve with nebulizer treatment given three times, need physician consultation. While oxygen administration and further nebulizer treatments may be indicated, it is incorrect to continue to monitor the patient without notifying the physician. 2. Which symptom in a patient diagnosed with asthma indicates severe bronchospasm? a. Breathlessness with minimal activity or eating b. Pausing to breathe while attempting to talk c. Repetitive, spasmodic coughing at night d. Wheezing after exposure to a trigger ANS: B Inability to speak a full sentence without pausing to breathe indicates severe bronchospasm. Breathlessness, repetitive and spasmodic coughing, and wheezing are all common signs of bronchospasm and do not necessarily indicate severe bronchospasm. MULTIPLE RESPONSE 1. Which clinical findings are worrisome in a patient experiencing acute bronchospasm, requiring immediate treatment? (Select all that apply.) a. A silent chest after previously wheezing b. Decreasing blood pressure c. Presence of an urticarial rash d. Pulsus paradoxus of 10 mm Hg e. Wheezing on both inspiration and expiration ANS: A, B, C A silent chest indicates severe spasm and is an ominous sign. Decreasing blood pressure and urticarial rash are present with anaphylaxis, which is a respiratory emergency requiring oxygen, diphenhydramine or epinephrine. A pulsus paradoxus greater than 25 mm Hg is worrisome. Wheezing on inspiration and expiration is a common finding and not necessarily an emergency. WWW.THENURSINGMASTERY.COM Downloaded by: dwadsw2 | [email protected] Distribution of this document is illegal Want to earn $1.236 extra per year? Stuvia.com - The Marketplace to Buy and Sell your Study Material Chapter 23: Anaphylaxis Buttaro: Primary Care: A Collaborative Practice, 6th Edition MULTIPLE CHOICE 1. A child with no previous history of asthma is brought to the emergency department with wheezing, stridor, and shortness of breath. When the child is started on oxygen and given a nebulized bronchodilator treatment, the treatment team notes a wheal and flare rash on the child’s trunk. What medication will be given immediately? a. Inhaled racemic epinephrine b. Intramuscular epinephrine c. Intravenous diphenhydramine d. Intravenous ranitidine ANS: B The patient has signs of anaphylaxis and should be given IM or SC epinephrine immediately as first-line therapy, with this repeated every 5 to 20 minutes as needed to prevent cardiovascular shock. Inhaled epinephrine is used for acute upper airway bronchospasm. Diphenhydramine and ranitidine are given as second-line treatment after epinephrine is administered or for mild, non-life-threatening allergic reactions. 2. A man self-administers aqueous epinephrine after experiencing a bee sting and developing angioedema and wheezing. What should the man do next? a. Obtain transport to an emergency department immediately. b. Repeat the epinephrine dose if needed and notify a physician of the episode. c. Resume normal activity if symptom free after 30 to 60 minutes. d. Take oral diphenhydramine and report any symptoms to a provider. ANS: A The man has a history of anaphylaxis and experienced symptoms after contact with a trigger. The aqueous epinephrine should be used immediately but does not prevent the need for follow up in an emergency department for close observation, since continued reaction to the allergen can occur for 6 to 8 hours. The epinephrine dose may be given if needed before emergency personnel arrive, but a second dose is not enough to prevent ongoing reaction to the allergen. WWW.THENURSINGMASTERY.COM Downloaded by: dwadsw2 | [email protected] Distribution of this document is illegal Want to earn $1.236 extra per year? Stuvia.com - The Marketplace to Buy and Sell your Study Material Chapter 24: Bites and Stings Buttaro: Primary Care: A Collaborative Practice, 6th Edition MULTIPLE CHOICE 1. A child experiences a snake bite while camping and is seen in the emergency department. The child’s parents are not able to identify the type of snake. An inspection of the site reveals two puncture wounds on the child’s arm with no swelling or erythema at the site. The child has normal vital signs. Which treatment is indicated? a. Administering antivenom and observing the child for 24 to 48 hours b. Cleaning the wound, giving tetanus prophylaxis, and observing for 12 hours c. Performing a type and cross match of the child’s blood d. Referral to a surgeon for incision and suction of the wound ANS: B The child does not have immediate symptoms of envenomation, since there is no swelling or erythema. Because symptoms may be delayed, and the type of snake is unknown, the child should be observed in an ED or hospital for 12 hours after providing wound care and tetanus prophylaxis. Antivenom is not indicated unless envenomation occurs. Type and cross match is done if envenomation is severe. Incision and suction of the sound is not recommended. MULTIPLE RESPONSE 1. A patient is seen in the emergency department after experiencing a spider bite. The spider is in a jar and is less than one inch in size, yellow-brown, and has a violin-shaped marking on its back. Depending on the patient’s symptoms, which treatments and diagnostic evaluations may be ordered? (Select all that apply.) a. Airway management b. An acute abdominal series c. Antivenom therapy d. CBC, BUN, electrolytes, and creatinine e. Coagulation studies f. Tetanus prophylaxis ANS: D, E, F The spider is a brown recluse. If the patient exhibits systemic symptoms, laboratory workup, including CBC, BUN, creatinine, electrolytes, and coagulation studies should be performed. Tetanus prophylaxis is given. Airway management, an acute abdominal series, and antivenom therapy are used for black widow spider bites. WWW.THENURSINGMASTERY.COM Downloaded by: dwadsw2 | [email protected] Distribution of this document is illegal Want to earn $1.236 extra per year? Stuvia.com - The Marketplace to Buy and Sell your Study Material Chapter 25: Bradycardia and Tachycardia Buttaro: Primary Care: A Collaborative Practice, 6th Edition MULTIPLE CHOICE 1. A patient prescribed a beta blocker medication is in the emergency department with reports of syncope, shortness of breath, and hypotension. A cardiac monitor reveals a heart rate of 35 beats per minute. Which medication may be used to stabilize this patient? a. Adenosine b. Amiodarone c. Atropine d. Epinephrine ANS: D Epinephrine is indicated if unstable bradycardia is caused by beta blockers. This patient is symptomatic and unstable and should be treated. Adenosine and amiodarone are used to treat tachycardia. Atropine is used for some types of bradycardia, but not when induced by beta blockers. 2. A patient reports heart palpitation but no other symptoms and has no prior history of cardiovascular disease. The clinic provider performs an electrocardiogram and notes atrial fibrillation and a heart rate of 120 beats per minute. Which is the initial course of action in treating this patient? a. Administer atenolol intravenously. b. Admit to the hospital for urgent cardioversion. c. Refer the patient to a cardiologist. d. Initiate steps to begin anticoagulant therapy. ANS: C This patient has no history of serious heart disease and does not have symptoms of chest pressure, acute MI, or congestive heart failure and may be referred to a cardiologist for evaluation and treatment but anticoagulant therapy to minimize the risk of clot formation should be started initially. The 2014 AHA Guidelines for Atrial Fibrillation recommend shared decision-making in regard to anticoagulation based on relative risk of the patient for thromboembolic event. Atenolol is given IV for patients who are unstable; the advanced life support treatment guidelines do not recommend treatment of tachycardia if the patient is stable. Urgent cardioversion is rarely needed if the heart rate is less than 150 beats per minute unless there are underlying heart conditions. 3. Which cardiac arrhythmia in an unstable patient requires unsynchronized shocks, or defibrillation? a. Atrial fibrillation b. Atrial flutter c. Monomorphic ventricular tachycardia d. Polymorphic ventricular tachycardia ANS: D Polymorphic ventricular tachycardia should be treated as ventricular fibrillation with unsynchronized shocks. The other arrhythmias are treated with synchronized cardioversion. WWW.THENURSINGMASTERY.COM Downloaded by: dwadsw2 | [email protected] Distribution of this document is illegal Want to earn $1.236 extra per year? Stuvia.com - The Marketplace to Buy and Sell your Study Material Chapter 26: Chemical Exposure Buttaro: Primary Care: A Collaborative Practice, 6th Edition MULTIPLE CHOICE 1. What is true when considering activated charcoal for gastrointestinal decontamination to treat a toxic substance ingestion? a. It acts by enhancing gastric motility to reduce absorption. b. It is administered only through a nasogastric tube. c. It may be used when petroleum distillates are ingested. d. Its use is controversial, though in specific situations can be used. ANS: D The use of activate charcoal is controversial, though in specific situations can be used for gastrointestinal decontamination. It absorbs ingested substances and reduces absorption and may cause bowel obstruction; it does not increase bowel motility. It may be given orally or by nasogastric tube. Because it is associated with vomiting, it should not be used when caustic substances, alcohols, and petroleum distillates are ingested. 2. A lawn maintenance worker is brought to the emergency department after an accident in which a large amount of pesticide was sprayed all over his clothing. He can relate the details of the accident to the emergency department personnel. What is the priority treatment on admission? a. Administer intravenous diphenhydramine and possibly epinephrine. b. Contact the Poison Control center to ask about appropriate antidotes. c. Place on a cardiorespiratory monitor and establish intravenous access. d. Remove the patient’s clothing and irrigate the skin for 15 to 30 minutes. ANS: D Most skin exposure to chemicals must be treated immediately with copious irrigation with water, so this is the initial priority in a stable patient. Since he can converse with staff, he is likely to be stable. If signs of anaphylaxis occur, diphenhydramine and epinephrine are indicated. The Poison Control center should be contacted, but this is not the priority. After irrigation to minimize exposure, other interventions, such as cardiorespiratory monitoring and IV access, may be necessary. MULTIPLE RESPONSE 1. A child is brought to the emergency department (ED) when a grandparent suspects ingestion of a tricyclic antidepressant medication found in the bathroom. What symptoms will the ED professionals expect to observe if this is the case? (Select all that apply.) a. Excessive salivation b. Flushed skin c. Hallucinations d. Hypothermia e. Mydriasis f. Urinary frequency ANS: B, C, E WWW.THENURSINGMASTERY.COM Downloaded by: dwadsw2 | [email protected] Distribution of this document is illegal Want to earn $1.236 extra per year? Stuvia.com - The Marketplace to Buy and Sell your Study Material Tricyclic antidepressants will cause anticholinergic effects, including flushing of the skin, hallucinations or psychosis, and mydriasis. These medications also cause dry mucous membranes, hyperthermia, and urinary retention. WWW.THENURSINGMASTERY.COM Downloaded by: dwadsw2 | [email protected] Distribution of this document is illegal Want to earn $1.236 extra per year? Stuvia.com - The Marketplace to Buy and Sell your Study Material Chapter 27: Electrical Injuries Buttaro: Primary Care: A Collaborative Practice, 6th Edition MULTIPLE CHOICE 1. An adolescent male has received an electrical injury from a high-voltage wire that was found lying on the ground. The adolescent is stabilized by the emergency medical service (EMS) personnel who responded to the call. Upon arrival at the emergency department, which diagnostic test is the priority? a. 12-lead electrocardiogram b. Cervical spine radiography c. Complete blood count and electrolytes d. Creatine kinase and myoglobin level ANS: A An early essential assessment in all patients with electrical injury is a 12-lead ECG to assess arrhythmias and conduction disturbances. The other labs are part of the initial workup, but not a priority over the ECG. A C-spine radiograph is done if cervical injury is suspected. MULTIPLE RESPONSE 1. What is true about electrical injuries? (Select all that apply.) a. Alternating current causes tetanic skeletal muscle contractions. b. Direct current is more dangerous than alternating current. c. Electrical injury causes more tissue necrosis in nerves than other tissues. d. Lightning is less lethal because the duration of electrical strike is short. e. Low-voltage contact has no potential to be lethal. ANS: A, C, D Alternating current tends to be more lethal than direct current because it causes tetanic muscle contractions. Electrical injury affects nerves more than other tissues because nerve tissue has the least resistance to direct flow and is most easily damaged. Lightning, although it has a voltage of 10 million to 2 billion volts, has a short duration of contact. Alternating current is more dangerous than direct current. Low-voltage contact has the potential to be lethal. WWW.THENURSINGMASTERY.COM Downloaded by: dwadsw2 | [email protected] Distribution of this document is illegal Want to earn $1.236 extra per year? Stuvia.com - The Marketplace to Buy and Sell your Study Material Chapter 28: Environmental and Food Allergies Buttaro: Primary Care: A Collaborative Practice, 6th Edition MULTIPLE CHOICE 1. Which immunoglobulin is responsible for initiating the allergic cascade in susceptible individuals who are exposed to allergens? a. IgG b. IgA c. IgM d. IgE ANS: D While IgA, IgG, and IgM are produced to appropriately protect the body, circulating levels of IgE are responsible for the atopic reaction. MULTIPLE RESPONSE 1. When performing diagnostic tests to determine which environmental allergens cause symptoms in an atopic patient, which aspects of scratch testing are preferable to other methods? (Select all that apply.) a. It has a lower potential for anaphylaxis. b. It is more sensitive. c. It is safer. d. It produces more rapid results. e. It requires a stepwise approach. ANS: A, C, D Scratch testing involves scratching the surface of the skin. This method has a lower potential for anaphylaxis, is safer, and has more rapid results. It is not as sensitive as the intradermal method, which requires a stepwise approach. 2. Which food allergies in children may be outgrown in the first decade of life? (Select all that apply.) a. Egg allergy b. Fish allergy c. Milk allergy d. Nut allergy e. Shell fish allergy ANS: A, C Both egg and milk allergy may be outgrown within the first decade of life. Fish, nut, and shell fish allergies are more common in adults and have a higher incidence of lifetime allergy. WWW.THENURSINGMASTERY.COM Downloaded by: dwadsw2 | [email protected] Distribution of this document is illegal Want to earn $1.236 extra per year? Stuvia.com - The Marketplace to Buy and Sell your Study Material Chapter 29: Head Trauma Buttaro: Primary Care: A Collaborative Practice, 6thEdition MULTIPLE CHOICE 1. A patient is brought to the emergency department after being hit in the head with a baseball. The patient is awake and talking but is confused and disoriented and does not obey simple commands. The patient can point to the area of pain and opens eyes only when commanded to do so. Bystanders report a period of unconsciousness lasting almost 5 minutes. Which severity of traumatic brain injury is likely? a. Normal b. Mild c. Moderate d. Severe ANS: C This patient’s Glasgow Coma score (GCS) is 11, based on eye opening to verbal command (3), ability to localize pain (4), and conversing while confused (4). The patient was unconscious less than 10 minutes, which usually indicates less severe injury. A patient with a GCS between 9 and 12 with or without loss of consciousness is considered to have a moderate head injury. 2. A patient is in the emergency department after sustaining a blow to the head in a motor vehicle accident. The patient’s Glasgow Coma score (GCS) is 14 and the patient is drowsy. The patient has a small amount of blood in one external auditory canal. Which is a priority in diagnosing the extent of injury in this patient? a. Close monitoring of pulse, respiration, and oxygenation b. Continued assessment of neurological status c. Magnetic resonance imaging of the head d. Non-enhanced computed tomography of the head ANS: D Although this patient’s GCS is nonconcerning, the type of injury and the sign of blood in the external auditory canal put this patient at high risk for skull fracture, so a head CT is indicated immediately. Close monitoring of vital signs and neurological status should be continuously performed, the CT is a priority to help determine the treatment needed. MRI is not especially useful but may be performed after CT if more detail of structures is needed. 3. A patient who sustained a head injury has a Glasgow Coma score (GCS) of 14. The patient’s spouse reported that the patient lost consciousness for approximately 7 minutes after falling down the stairs. A head computed tomography (CT) scan does not reveal brain lesions. Which treatment is indicated? a. Admission to the hospital with a neurosurgical evaluation b. Continued observation in the emergency department until stability is ensured c. Discharge to home with close observation by the patient’s spouse for 24 hours d. Dismissal to home with a referral for follow-up with a neurologist ANS: A WWW.THENURSINGMASTERY.COM Downloaded by: dwadsw2 | [email protected] Distribution of this document is illegal Want to earn $1.236 extra per year? Stuvia.com - The Marketplace to Buy and Sell your Study Material This patient had loss of consciousness longer than 5 minutes and has a GCS of 14; both are indications for admission to the hospital with a neurosurgery consult, even though the CT is currently normal. WWW.THENURSINGMASTERY.COM Downloaded by: dwadsw2 | [email protected] Distribution of this document is illegal Want to earn $1.236 extra per year? Stuvia.com - The Marketplace to Buy and Sell your Study Material Chapter 30: Hypotension Buttaro: Primary Care: A Collaborative Practice, 6th Edition MULTIPLE CHOICE 1. A young adult patient is being treated for hypertension and is noted to have a resting blood pressure of 135/88 mm Hg just after finishing a meal. After standing, the patient has a blood pressure of 115/70 mm Hg. What is the likely cause of this change in blood pressure? a. A hyperglycemic episode b. Antihypertensive medications c. Neurogenic orthostatic hypotension d. Postprandial hypotension ANS: B Medications to treat hypertension may cause orthostatic hypotension. Hypoglycemia may cause hypotension. Neurogenic orthostatic hypotension is less likely since there is no direct connection to the neurological system. Postprandial hypotension occurs in elderly patients. 2. An elderly patient who experiences orthostatic hypotension secondary to antihypertensive medications is noted to have a drop in systolic blood pressure of 25 mm Hg. Which intervention is important for this patient? a. Administration of intravenous fluids b. Close monitoring cardiorespiratory status c. Initiation of a fall risk protocol d. Withholding antihypertensive medications ANS: C A reduction of systolic blood pressure >20 mm Hg is a risk factor for falls in the elderly, so a fall risk protocol should be initiated. Unless the patient is dehydrated, IV fluids are not recommended. Close monitoring of CR status will not prevent falls. Withholding antihypertensive medications often worsens orthostatic hypotension. 3. An older patient develops orthostatic hypotension secondary to an antihypertensive medication and asks what measures can be taken to minimize this condition. What will the provider recommend? a. Changing from sitting to standing slowly b. Decreasing the medication dosage c. Decreasing physical activity d. Performing the Valsalva maneuver ANS: A Changing positions slowly will assist in minimizing the effects of this condition. Decreasing activity will not help and will have a negative effect on general health. Decreasing or discontinuation of the medication should not be done without first contracting the prescribing health care provider. Performing the Valsalva maneuver will increase intrathoracic pressure and should be avoided. WWW.THENURSINGMASTERY.COM Downloaded by: dwadsw2 | [email protected] Distribution of this document is illegal Want to earn $1.236 extra per year? Stuvia.com - The Marketplace to Buy and Sell your Study Material Chapter 31: Poisoning Buttaro: Primary Care: A Collaborative Practice, 6th Edition MULTIPLE CHOICE 1. During chemistry class, a nursing student has accidentally splashed a chemical into his or her left eye. What intervention has priority while awaiting the arrival of the emergency medical service (EMS) personnel? a. Flushing the eye with water b. Removing the student to a quiet, dark area c. Encouraging the student to relax and remain calm d. Securing a sample of the chemical to be given to EMS ANS: A In general, skin and eye decontamination are done immediately on hospital arrival (should be started prehospital, if possible, and completed on hospital arrival). While encouraging the student to remain calm and placing them in a quiet environment as well as preparing to provide a sample of the chemical may all be helpful, none of those interventions have priority over beginning the decontamination of the eye. 2. A patient who ingested a bottle of acetaminophen tablets is brought to the emergency department. Which treatment is indicated? a. Flumazenil b. N-acetylcysteine c. Naloxone d. Supportive care only ANS: B N-acetylcysteine is used as an antidote for acetaminophen overdose. Flumazenil is used to treat benzodiazepine overdose. Naloxone is given for opioid overdose. 3. What is the priority in emergency management of a biological terrorism attack? a. Basic life support b. Communication with authorities c. Containing the exposures d. Informing the public of the risk ANS: C In a bioterrorism attack, the initial priority is to contain the exposures and prevent expansion of the event to others. Basic life support is the second priority and close communication with authorities is the third priority. Informing the public is a later priority after the situation has been stabilized. WWW.THENURSINGMASTERY.COM Downloaded by: dwadsw2 | [email protected] Distribution of this document is illegal Want to earn $1.236 extra per year? Stuvia.com - The Marketplace to Buy and Sell your Study Material Chapter 32: Sexual Assault Buttaro: Primary Care: A Collaborative Practice, 6th Edition MULTIPLE CHOICE 1. When beginning a health maintenance exam, the health care provider learns that an adult patient has been sexually assaulted the previous day. What is the initial responsibility of the provider? a. Notify the police and encourage the patient to press criminal charges. b. Perform a thorough gynecological exam and obtain cultures. c. Question the patient about the events surrounding the assault. d. Refer the patient to the emergency department (ED) for a forensic examination. ANS: D If a patient has been sexually assaulted within the past 5 days, and especially if within the previous 72 hours, the provider should defer a physical examination and refer the patient to the ED for a forensic examination. It is not necessary to notify the police unless the victim is a child, elderly, or disabled. The provider should not perform the exam—a forensic exam ensures that standard protocol is followed, and appropriate evidence is obtained. Retelling the story of the assault may be traumatizing to the patient, so this should be left to providers performing the forensic exam. 2. A patient who was sexually assaulted one month prior tells her provider that she is concerned about contracting human immunodeficiency virus (HIV). When is it appropriate to perform testing? a. Immediately and then every 6 months for the first year b. Immediately with definitive results c. In 2 weeks and then 3 to 6 months after the assault d. Three to six months after the assault ANS: C Because of the length of time for seroconversion to occur, patients concerned about HIV exposure should be tested 6 weeks after and then 3 to 6 months after the assault. Immediate results will not provide accurate information. The initial testing should be 6 weeks after potential exposure. 3. During a health maintenance examination, 17-year-old female reports having been raped repeatedly at a college party during the previous semester and tells the practitioner that she did not seek help at the time. Which action is a priority for the primary care provider? a. Recommending counseling at a local mental health center b. Referring the patient to the emergency department for sexually transmitted infection (STI) testing c. Reporting the alleged assault to law enforcement d. Suggesting that the patient report the incident to the school ANS: C Any sexual assault perpetrated on a victim younger than 18 years must be reported to the local child or adult protective agency as well as to law enforcement, regardless of whether the patient reports that sexual assault occurred. Counseling, STI testing, and reporting the incident to the school are important, but are not the priority. WWW.THENURSINGMASTERY.COM Downloaded by: dwadsw2 | [email protected] Distribution of this document is illegal Want to earn $1.236 extra per year? Stuvia.com - The Marketplace to Buy and Sell your Study Material Chapter 33: Syncope Buttaro: Primary Care: A Collaborative Practice, 6th Edition MULTIPLE CHOICE 1. A healthy 20-year-old patient reports having had 1 or 2 episodes of syncope without loss of consciousness. Which is the most likely type of syncope in this patient? a. Cardiac b. Neurogenic c. Orthostatic hypotensive d. Reflex syncope ANS: D Neurally mediated or reflex syncope is the most common cause of syncope and is primarily seen in young adults. Cardiac, neurogenic, and orthostatic syncope are generally seen in older adults. 2. An elderly patient reports experiencing syncope each morning when getting out of bed. Which assessment will the health care provider perform first to evaluate this patient’s symptoms? a. Cardiac enzyme levels b. Electroencephalogram c. Fasting blood glucose d. Orthostatic blood pressures ANS: D Orthostatic blood pressures should be measured first since this patient reports problems associated with rising from a supine position. The other tests are performed as part of the diagnostic workup only if indicated by associated symptoms or suspected causes. MULTIPLE RESPONSE 1. Which tests are indicated as part of the initial evaluation for women of childbearing age who report syncope? (Select all that apply.) a. 12-lead electrocardiogram b. Cardiac enzyme levels c. Complete blood count d. Electroencephalogram e. Serum glucose testing ANS: A, C, E Initial evaluation for all patients reporting syncope should include a standard 12-lead ECG. Women of childbearing age should have a CBC, serum pregnancy test, and serum glucose testing. Cardiac enzyme levels are obtained if the patient has cardiac risk factors. EEG is performed only if there is a concern for seizure disorder. WWW.THENURSINGMASTERY.COM Downloaded by: dwadsw2 | [email protected] Distribution of this document is illegal Want to earn $1.236 extra per year? Stuvia.com - The Marketplace to Buy and Sell your Study Material Chapter 34: Thermal Injuries Buttaro: Primary Care: A Collaborative Practice, 6th Edition MULTIPLE CHOICE 1. A provider attending a soccer match on a hot day is assisting a player who feels hot and appears dehydrated, but who is alert and oriented. What does the provider suspect? a. Heat cramps b. Heat exhaustion c. Heat stroke d. Heat syncope ANS: B Heat exhaustion is present when patients have excessive sweating accompanied by sodium and water loss. Heat cramps involve muscle pains or spasms. Heat stroke causes a core body temperature of >106°F. Heat syncope causes fainting or dizziness. 2. A patient is brought to a clinic after fainting while working outdoors on a hot day. The patient has slurred speech and headache and has a temperature of 104°F. What will the provider do? a. Administer antipyretic medications to reduce the temperature. b. Administer intravenous fluids in the clinic and monitoring response. c. Rehydrate the patient with oral fluids containing electrolytes. d. Transport the patient to the emergency department (ED). ANS: D This patient has CNS signs and an elevated temperature with a history consistent with heat stroke. This patient should be immediately transported to an emergency department. Antipyretic medications are not useful for treating thermal injury. The patient will be given IV fluids and electrolytes in the ED. Oral rehydration is not indicated. MULTIPLE RESPONSE 1. A child is brought to the emergency department after getting lost while camping on a cold, rainy day. The child is lethargic on admission. The cardiorespiratory monitor shows a normal heart rate and rhythm, a respiratory rate of 8 to 10 breaths per minute, and a normal blood pressure. The assessment reveals erythema and edema of the child’s hands and feet. What treatments are indicated? (Select all that apply.) a. Administer antibiotics. b. Apply warmed blankets. c. Elevate the child’s extremities. d. Massage the hands and feet. e. Remove all wet clothing. ANS: B, C, E The child has signs of frostbite without other systemic signs. Warming with warm blankets is indicated. The affected areas should be elevated, but not massaged or rubbed. The providers should remove the child’s clothing which may be restrictive or wet and examine the child’s entire skin surface for other signs of frostbite. Antibiotics are not given unless signs of infection are present. WWW.THENURSINGMASTERY.COM Downloaded by: dwadsw2 | [email protected] Distribution of this document is illegal Want to earn $1.236 extra per year? Stuvia.com - The Marketplace to Buy and Sell your Study Material Chapter 35: Examination of the Skin and Approach to Diagnosis of Skin Disorders Buttaro: Primary Care: A Collaborative Practice, 6th Edition MULTIPLE CHOICE 1. A primary care provider is performing a Tzanck test to evaluate possible herpes simplex lesions. To attain accurate results, the provider will perform what intervention? a. Blanch the lesions while examining them with a magnifying glass. b. Gently scrape the lesions with a scalpel onto a slide. c. Perform a gram stain of exudate from the lesions. d. Remove the top of the vesicles and obtain fluid from the lesions. ANS: D The Tzanck test requires removing the tops from vesicular lesions in order to obtain fresh fluid from the base of the lesions. Blanching of blue to red lesions under a microscope helps to evaluate whether blood is in the capillaries of the lesions. Scraping lesions onto a slide is done to evaluate the presence of hyphae and spores common with candidiasis or fungal infections. Gram staining is performed to distinguish gram-positive from gram-negative organisms in suspected bacterial infections. MULTIPLE RESPONSE 1. When examining a patient’s skin, a practitioner uses dermoscopy for what purpose? (Select all that apply.) a. Accentuating changes in color of pathologic lesions by fluorescence b. Assessing changes in pigmentation throughout various lesions c. Determining whether lesion borders are regular or irregular d. Differentiating fluid masses from cystic masses in the epidermis e. Visualizing skin fissures, hair follicles, and pores in lesions ANS: B, C, E Dermoscopy is used to visualize the epidermis and superficial dermis and can reveal changes in pigmentation throughout lesions, whether borders are regular or irregular, and the various fissures, follicles, and pores present in lesions. The Wood’s light, or black light, is used to fluoresce lesions to accentuate changes in color. A direct light source is useful for differentiating fluid masses from cystic masses. WWW.THENURSINGMASTERY.COM Downloaded by: dwadsw2 | [email protected] Distribution of this document is illegal Want to earn $1.236 extra per year? Stuvia.com - The Marketplace to Buy and Sell your Study Material Chapter 36: Surgical Office Procedures Buttaro: Primary Care: A Collaborative Practice, 6th Edition MULTIPLE CHOICE 1. A patient has actinic keratosis and the provider elects to use cryosurgery to remove the lesions. How will the provider administer this procedure? a. Applying one or two freeze-thaw cycles to each lesion b. Applying two or more freeze-thaw cycles to each lesion c. Applying until the freeze spreads laterally 1 mm from the lesion edges d. Applying until the freeze spreads laterally 4 mm from the lesion edges ANS: A For actinic keratosis, one to two freeze-thaw cycles are usually enough. Two or more freeze-thaw cycles are generally required for thicker, seborrheic keratosis lesions. The freeze should spread laterally 3 to 4 mm from the edge of the lesions. 2. A provider is preparing to administer electrocautery to a patient who has several seborrheic keratoses. The patient tells the provider that he has a pacemaker. Which action is correct? a. Administer the electrocautery per the usual protocol. b. Apply electrocautery in short burst at low voltage. c. Refer the patient to a dermatologist for removal. d. Suggest another method for removal of the lesions. ANS: B Patients with pacemakers or implantable cardioverter-defibrillators may receive electrocautery if appropriate precautions, such as lower voltage and shorter bursts are taken. It is not necessary to suggest another method or to refer to a dermatologist. 3. Which type of office surgical procedure warrants sterile technique? a. Curettage b. Punch biopsy c. Scissor excision d. Shave biopsy ANS: B Punch biopsy requires sterile technique. The other procedures require cleaning with alcohol and clean technique with universal precautions. WWW.THENURSINGMASTERY.COM Downloaded by: dwadsw2 | [email protected] Distribution of this document is illegal Want to earn $1.236 extra per year? Stuvia.com - The Marketplace to Buy and Sell your Study Material Chapter 37: Principles of Dermatologic Therapy Buttaro: Primary Care: A Collaborative Practice, 6th Edition MULTIPLE CHOICE 1. When recommending an over-the-counter topical medication to treat a dermatologic condition, which instruction to the patient is important to enhance absorption of the drug? a. Apply a thick layer of medication over the affected area. b. A solution spray preparation will be more effective on hairy areas. c. Put cool compresses over the affected area after application. d. Use a lotion or cream instead of an ointment preparation. ANS: B Hairy areas are difficult to penetrate, so in these areas, a solution, foam, spray, or gel may work better. Applying a thicker layer does not increase skin penetration or effectiveness of a medication. Warm or inflamed skin absorbs medications more readily; cool compresses will decrease absorption. Lotions and creams are not as readily absorbed as ointments, which have occlusive properties. 2. A provider is prescribing a topical dermatologic medication for a patient who has open lesions on a hairy area of the body. Which vehicle type will the provider choose when prescribing this medication? a. Cream b. Gel c. Ointment d. Powder ANS: B Gels are an excellent vehicle for use on hairy areas of the body. Creams and ointments are not recommended for hairy areas. Powders should be avoided in open wounds. 3. An adult patient has been diagnosed with atopic dermatitis and seborrheic dermatitis with lesions on the forehead and along the scalp line. Which is correct when prescribing a corticosteroid medication to treat this condition? a. Initiate treatment with 0.1% triamcinolone acetonide. b. Monitor the patient closely for systemic adverse effects during use. c. Place an occlusive dressing over the medication after application. d. Prescribe 0.05% fluocinonide to apply liberally. ANS: A Treatment with 0.1% triamcinolone acetonide is appropriate in this case, because it is a class 4 corticosteroid and may be used on the face and is suggested for use for these conditions. Systemic side effects are rare when topical corticosteroids are used appropriately. Occlusive dressings increase the risk of adverse effects and are not recommended. 0.05% fluocinonide is a class III corticosteroid and should not be used on the face. WWW.THENURSINGMASTERY.COM Downloaded by: dwadsw2 | [email protected] Distribution of this document is illegal Want to earn $1.236 extra per year? Stuvia.com - The Marketplace to Buy and Sell your Study Material Chapter 38: Screening for Skin Cancer Buttaro: Primary Care: A Collaborative Practice, 6th Edition MULTIPLE CHOICE 1. During a total body skin examination for skin cancer, the provider notes a raised, shiny, slightly pigmented lesion on the patient’s nose. What will the provider do? a. Consult with a dermatologist about possible melanoma. b. Reassure the patient that this is a benign lesion. c. Refer the patient for possible electrodessication and curettage. d. Tell the patient this is likely a squamous cell carcinoma. ANS: C This lesion is characteristic of basal cell carcinoma, which is treated with electrodessication and curettage. Melanoma lesions are usually asymmetric lesions with irregular borders, variable coloration, >6 mm diameter, which are elevated; these should be referred immediately. All suspicious lesions should be biopsied; until the results are known, the provider should not reassure the patient that the lesion is benign. Squamous cell carcinoma is roughened, scaling, and bleeds easily. 2. What is the initial approach when obtaining a biopsy of a potential malignant melanoma lesion? a. Excisional biopsy b. Punch biopsy c. Shave biopsy d. Wide excision ANS: A A suspected malignant melanoma lesion should be biopsied with excisional biopsy; if diagnosed, a wide excision should follow. Punch and shave biopsy procedures are appropriate for diagnostic evaluation of NMSC lesions. WWW.THENURSINGMASTERY.COM Downloaded by: dwadsw2 | [email protected] Distribution of this document is illegal Want to earn $1.236 extra per year? Stuvia.com - The Marketplace to Buy and Sell your Study Material Chapter 39: Adnexal Disease Buttaro: Primary Care: A Collaborative Practice, 6th Edition MULTIPLE CHOICE 1. A patient has acne and the provider notes lesions on half of the face, some nodules, and two scarred areas. Which treatment will be prescribed? a. Oral clindamycin for 6 to 8 weeks b. Oral isotretinoin c. To

Use Quizgecko on...
Browser
Browser